If $\Phi\geq 0$ Is Non-decreasing, Does $\int_1^\infty \frac{\Phi(x)}{x ...
Có thể bạn quan tâm
-
- Home
- Questions
- Tags
- Users
- Unanswered
- Teams
Ask questions, find answers and collaborate at work with Stack Overflow for Teams.
Try Teams for free Explore Teams - Teams
-
Ask questions, find answers and collaborate at work with Stack Overflow for Teams. Explore Teams
Teams
Q&A for work
Connect and share knowledge within a single location that is structured and easy to search.
Learn more about Teams If $\Phi\geq 0$ is non-decreasing, does $\int_1^\infty \frac{\Phi(x)}{x^2}\,dx=\infty$ imply $\int_0^\infty e^{-\Phi(x)}\,dx<\infty$? Ask Question Asked 3 years, 4 months ago Modified 3 years, 4 months ago Viewed 634 times 24 $\begingroup$Q. Suppose $\Phi : [0, \infty) \to [0, \infty)$ is a non-decreasing function. If $$ \int_{1}^{\infty} \frac{\Phi(x)}{x^2} \, \mathrm{d}x = \infty, $$ then does the inequality $$ \int_{0}^{\infty} e^{-\Phi(x)} \, \mathrm{d}x < \infty $$ hold?
This question is motivated by this posting, but it became a statement of independent interest to me.
I suspect that the statement is true, based on a naive observation that a higher growth rate of $\Phi$ is in favor of both conditions and a slower growth rate is against them.
One obvious observation is that $\Phi(x)$ must tend to $\infty$ as $x \to \infty$. But, to be honest, I have no clue as to where I should start. So I would like to invite you to this strange yet interesting problem!
Edit. The user @Sungjin Kim showed that the answer is false by providing a family of counter-examples. Here is a simplification of his answer:
For each strictly increasing sequence $(b_k)_{k=1}^{\infty}$ in $[1, \infty)$, let
$$ \Phi(t) = \sum_{k \mathop{:} b_k \leq t} b_k = \sum_{k=1}^{\infty} b_k \mathbf{1}_{[b_k, \infty)}(t). $$
Then it follows that
$$ \int_{1}^{\infty} \frac{\Phi(t)}{t^2} \, \mathrm{d}t = \sum_{k=1}^{\infty} b_k \int_{b_k}^{\infty} \frac{\mathrm{d}t}{t^2} = \sum_{k=1}^{\infty} 1 = \infty. $$
On the other hand, since $\Phi(t) = \sum_{l=1}^{k} b_l$ for each $t \in [b_k, b_{k+1})$,
\begin{align*} \int_{0}^{\infty} e^{-\Phi(t)} \, \mathrm{d}t \geq \sum_{k=1}^{\infty} \int_{b_k}^{b_{k+1}} e^{-\Phi(t)} \, \mathrm{d}t = \sum_{k=1}^{\infty} (b_{k+1} - b_k) e^{-\sum_{l=1}^{k} b_l}. \end{align*}
So by choosing $(b_{k+1})$ so that it satisfies $(b_{k+1} - b_k) e^{-\sum_{l=1}^{k} b_l} \geq 1$ for each $k$, we have
$$ \int_{0}^{\infty} e^{-\Phi(t)} \, \mathrm{d}t = \infty. $$
Share Cite Follow edited Jul 25, 2021 at 15:54 Sangchul Lee asked Jul 22, 2021 at 20:06 Sangchul LeeSangchul Lee 174k17 gold badges286 silver badges460 bronze badges $\endgroup$ 4- $\begingroup$ As an observation, there seems to be a 'breakeven' point where the first and second integrals are both convergent. This happens for functions like $\Phi(x)=2\ln(x+1)$. Maybe someone can use this to get some bounds on what $\Phi(x)$ can actually be $\endgroup$ – QC_QAOA Commented Jul 22, 2021 at 21:25
- 1 $\begingroup$ Nice simplification! One typo is in $\Phi(t)=\sum_l b_k$. It should be $\sum_l b_l$. $\endgroup$ – Sungjin Kim Commented Jul 25, 2021 at 15:53
- 1 $\begingroup$ But what if we require $\Phi$ to be smooth? Can this counterexample be modified to smooth out the jumps? $\endgroup$ – Simon Parker Commented Jul 27, 2021 at 19:07
- 1 $\begingroup$ @SimonParker, We may mollify $\Phi$ to be smooth. For example, we may mollify $\Phi$ to a smooth function $\tilde{\Phi}$ such that $\Phi(x) \leq \tilde{\Phi}(x) \leq \Phi(x-1)$ and use $\tilde{\Phi}$ instead. A fairly simple comparison will then allow us to directly use our observation for $\Phi$. $\endgroup$ – Sangchul Lee Commented Jul 27, 2021 at 19:13
1 Answer
Sorted by: Reset to default Highest score (default) Date modified (newest first) Date created (oldest first) 12 $\begingroup$We may assume $\Phi(1)=0$. Let $\Phi(x)=\int_1^x f(t) dt$ for some nonnegative function $f$. By assumption, we have $$ \int_1^{\infty} \int_1^x \frac{f(t)}{x^2}dtdx =\int_1^{\infty}\int_t^{\infty} \frac{f(t)}{x^2} dxdt = \int_1^{\infty} \frac{f(t)}tdt=\infty. $$ Let $$ a_{\ell} = \int_{2^{\ell-1}}^{2^{\ell}} \frac{f(t)}t dt. $$ We have $\sum_{\ell=1}^{\infty} a_{\ell} = \infty$ and $$ \int_{2^{\ell-1}}^{2^{\ell}} f(t) dt \leq 2^{\ell}\int_{2^{\ell-1}}^{2^{\ell}} \frac{f(t)}t dt=2^{\ell}a_{\ell}. $$ On the other hand, by Cauchy condensation test, the convergence of $\int_1^{\infty} e^{-\Phi(x)} dx$ is equivalent to the convergence of LHS of $$ \sum_{k=1}^{\infty} 2^k e^{-\Phi(2^k)} \geq \sum_{k=1}^{\infty} 2^k e^{-\sum_{\ell\leq k} 2^{\ell}a_{\ell} }. $$ Let $a_{\ell}$ be a characteristic function of a very rapidly increasing sequence $\{b_n\}$ of natural numbers.
If $b_m\leq k < b_{m+1}$, we have $$ 2^k e^{-\sum_{\ell\leq k} 2^{\ell}a_{\ell} }\geq 2^{b_m} e^{-2^{1+b_m}}. $$ Then the sum over $k$ in this interval is $$ \geq (b_{m+1}-b_m)2^{b_m} e^{-2^{1+b_m}}. $$ For each $m$, we take $b_{m+1}$ large enough that $$ (b_{m+1}-b_m)2^{b_m} e^{-2^{1+b_m}}\geq 1. $$ Then we have the divergence of the series $$ \sum_{k=1}^{\infty} 2^k e^{-\sum_{\ell\leq k} 2^{\ell}a_{\ell} }. $$ Hence, we have the divergence of $$ \int_1^{\infty} e^{-\Phi(x)} dx. $$
Share Cite Follow answered Jul 25, 2021 at 14:33 Sungjin KimSungjin Kim 20.6k3 gold badges29 silver badges73 bronze badges $\endgroup$ 4- 1 $\begingroup$ I am not claiming the divergence in all case. I showed a counterexample. $\endgroup$ – Sungjin Kim Commented Jul 25, 2021 at 14:52
- 1 $\begingroup$ Great answer! It is interesting to have a counter-example. :) $\endgroup$ – Sangchul Lee Commented Jul 25, 2021 at 15:03
- $\begingroup$ I tried to simplify your answer by myself and updated my posting. Again, thank you for your elegant idea! $\endgroup$ – Sangchul Lee Commented Jul 25, 2021 at 15:09
- $\begingroup$ @SungjinKim Ahh sry, I misunderstood what you were doing. $\endgroup$ – Maximilian Janisch Commented Jul 25, 2021 at 16:13
You must log in to answer this question.
Not the answer you're looking for? Browse other questions tagged
.- Featured on Meta
- We’re (finally!) going to the cloud!
- More network sites to see advertising test [updated with phase 2]
Linked
10 Is $\int_{\mathbb{R}^2} e^{-u} \Delta u < \infty$?Related
3 On the absolute integrability of radially symmetric functions 1 Constructing a specific normalised test function. 5 On the integral $\int_{0}^{\pi/2} \frac{x \log \left ( 1-\sin x \right )}{\sin x} \, \mathrm{d}x$ 2 Prove $\sum_{n\le x} d(n)/n = \frac12 \log^2 x + 2e \log x + c + \mathcal O(1/\sqrt{x})$ where $d$ is the divisor function and $c$ is a constant 3 A counterexample on tails of distribution 2 Limits using layer-cake representations for $L^p$ space involving two functions 4 About an integral from the MIT integration bee 2022 1 Integrating $\int_{0}^{\infty} \mathrm{d}x \frac{x^{2 k +1}}{\cosh \left( x- y\right)} $Hot Network Questions
- Can Windows firewall WF.msc do TCP port forwarding at all, like Linux iptables does?
- Is ATL-98 Carvair still alive in the US?
- What happened to the lifeboats in Star Trek: First Contact?
- Simultaneously cooling and humidifying in winter?
- How is one supposed to play these notes?
- Non-reflexive use of laisser without a direct object in « The Stranger » ?
- What is the proper way to say "voice direction" in German?
- RegionIntersection of 3D objects
- Custom document class always pick the default logo even if I change to other logos
- Full Bridge Rectifier - Output Voltage Saturation
- Difference between English short stories and short English stories
- Spacesuit for Vermoid Locomotion
- Implicit function theorem without manifolds (Steve Smale article)?
- Use of “12 m.” for noon and “12 p.m.” for midnight
- Formative alternative to midterms for a large class
- will "brown aluminum" drip-edge suffer galvanic corrosion if it rests against fascia made of copper-treated lumber?
- How to identify unsafe trees for climbing stand?
- Can .zshrc be modified automatically by other programs, installers, etc.?
- Why is this soldering iron red hot in the middle section?
- Using Reverse Tunnel for accessing URL not directly accessible
- Can basic authentication somehow be made to remember the login when closing the browser?
- What's the contrary of formation as in the opposite of the process of formation?
- Why doesn't gas usually contract in volume?
- Baskervaldx doesn’t allow certain glyphs
To subscribe to this RSS feed, copy and paste this URL into your RSS reader.
Từ khóa » Dx Phi
-
Dive Into Dx: Phi - YouTube
-
Int E^x {f(x)-f'(x)}dx= Phi(x), Then Int E^x F(x) Dx Is - Doubtnut
-
By Substitution: Theorem: If Int F(x) Dx = Phi(x) Then Int F(ax+b) Dx = 1 ...
-
If D/(dx)(phi(x))=f(x), Then - Doubtnut
-
( \\int \\phi ( X ) \\phi ^ { \\prime } ( X ) E ^ { \\phi ( X ) } Dx \\) - Toppr
-
If φ(x) Is The Inverse Of The Function F(x) And F'(x) = 1/(1+x%5), Then D ...
-
PHI VILLA Patio Round Furniture Set Covers Waterproof, Outdoor ...
-
[PDF] Tables Of The Functions Integral-phi-0-sin1/3 X Dx And (4/3) Sin-4/3 ...
-
[Solved] If \(\int E^x (f(x) - F'(x))dx = \phi(x),\) Then T
-
How To Evaluate This Integral [math]\displaystyle\int_ {0} ^ {\infty} \frac ...
-
Các Quy định Hành Lý Xách Tay Và Hành Lý Ký Gửi
-
Integrate Sin X Dx From X=0 To Pi - Wolfram|Alpha
-
List Of Integrals Of Gaussian Functions - Wikipedia